Ethics EPPP Test Questions

Réussis tes devoirs et examens dès maintenant avec Quizwiz!

The difference between insanity and psychosis is that: A. insanity is a legal term and psychosis is a term used in mental health literature B. insanity is an outdated term and is no longer applied in legal cases, while psychosis is used both in mental health and legal literature C. insanity is legally a broader term, and subsumes the term psychosis D. insanity refers to disturbances in thought and emotion, while a psychotic disturbance, by definition, affects thought only

Correct Answer is: A Insanity is strictly a legal term; it usually means that a person is in such a mental state that he or she cannot distinguish between right or wrong. Psychosis, on the other hand, is a psychiatric term.

Although you have extensive experience in administering the WAIS-R, you have not yet administered the WAIS-III. If you would like to administer the WAIS-III you should: A. obtain additional training B. obtain supervision by a psychologist experienced in the use of the WAIS-III C. seek consultation to determine the most appropriate action to take D. be able to administer the WAIS-III since it is very similar to the WAIS-R

Correct Answer is: A Standard 2.01(c) (Boundaries of competence), requires psychologists who provide services involving populations, areas, techniques, or technologies that are new to them undertake relevant education, training, supervised experience, consultation, or study. When administering a test, or a new version of a test which you are not familiar with, the best course of action would be to obtain the necessary training.

The purpose of licensure in psychology and other mental health fields is to Select one: A. protect the consumer by limiting practice to those who are qualified. B. provide uniform national standards of minimal competence in the field. C. protect the public by ensuring that professional have enough knowledge about the field. D. allow consumers to choose professionals who are the best in the field.

Correct Answer is: A All 50 states in the U.S., the 10 provinces in Canada, and the territories of Guam, Puerto Rico, U.S. Virgin Islands, and the District of Columbia regulate the practice of psychology and require psychologists to obtain licenses in order to practice professionally. The purpose of licensure and licensure laws are to protect the public by only allowing qualified persons to practice; i.e., persons who have demonstrated that have met minimum standards of competence as defined by state or provincial law. provide uniform national standards of minimal competence in the field. Contrary to this option, licensure is a matter of state law, not national standards. protect the public by ensuring that professional have enough knowledge about the field. This choice is not complete enough; licensure is intended to ensure that professionals meet standards of competence, which extend beyond just knowledge.

You have been court-ordered to evaluate a prisoner who is being tried for murder. You explain the purpose of the evaluation and complete it. As you are packing up your testing materials, the prisoner smirks, and says, "I agreed to this but I didn't sign anything. You are out of luck, doctor." Select one: A. You should go ahead and complete the report, but leave out the prisoner's comments. B. The prisoner is correct -- you should refer for a new assessment. C. You should add this verbal comment to the report, noting the prisoner's passive-aggressive tendencies. D. You should have read the case file because you would have learned that the accused is also an attorney; this would affect how you would treat this prisoner.

Correct Answer is: A Because this evaluation was court-ordered you were not actually required to obtain informed consent. However Standard IV.E.1 of the Speciality Guidelines for Forensic Psychologists states that "if the client appears unwilling to proceed after receiving a thorough notification of the purposes, methods, and intended uses of the forensic evaluation, the psychologist should take steps to place the client in contact with his or her attorney for the purpose of legal advice on the issue of participation." Remember, you always want to take the most careful and conservative approach if possible. You should add this verbal comment to the report, noting the prisoner's passive-aggressive tendencies. While you may be tempted to choose this option and add the client's comments to your report, this would contradict Standard V.C. of the Forensic Guidelines which states that, "In situations where the right of the client to confidentiality is limited, the forensic psychologist makes every effort to maintain confidentiality with regard to any information that does not bear directly upon the legal purpose of the evaluation." You should have read the case file because you would have learned that the accused is also an attorney; this would affect how you would treat this prisoner. Finally, as a thorough evaluator you should have read the case file closely; however, you would not proceed with an evaluation differently just because your client is an attorney.

A new client tells you that she recently had sex with her previous therapist. She wants you to "help her put the experience behind her". You should: A. Report this situation immediately B. Not report, but counsel her about the process C. Call the offending party but not give the client's name D. Encourage her to recall the event so she doesn't suffer from repression later

Correct Answer is: B You are obligated to protect your client's confidentiality. However you want to educate her about the Ethical Standards regarding this situation and also explain to her avenues of recourse she could take.

A man contacts a psychologist via e-mail, asking her to provide him with online therapy advising him about a decision that is weighing heavily on him. The man writes making decisions is often hard for him and he is trying to decide whether to quit his current job and accept an offer from a competing company. The man doesn't want to reveal his name to the psychologist. He wants to keep his identity private. The psychologist should: Select one: A. let the man remain anonymous if he wants and provide him with online therapy B. let the man know that before providing any online therapy to him, she has to have basic contact information in case of an emergency such as suicidality C. let the man know that, by law, she has to have a face-to-face meeting with him before she can provide him with online therapy D. let the man know that, by law, she has to know his name and some basic contact information before she can provide him with online therapy

Correct Answer is: A E-therapy refers to the delivery of mental health services online, usually delivered in the form of email communications, discussion lists, live chat rooms, or live audio or audiovisual conferencing and may include "coaching" , psychoeducation, emotional support, interactive journaling, etc. While many e-therapists like to have their clients' names and contact information in case of an emergency, and some even require this, it is both legal and ethical to allow a client receiving e-therapy to remain anonymous if that's what he or she wants. An example of a parallel would be with an anonymous patient who calls a hotline or an emergency room. Like many e-therapists, this psychologist could start by letting this client remain anonymous if he insists and then, as trust develops between them, gradually convince him to provide his name and contact information. (Practically speaking, the client identity problem will be solved online to a large extent as it is in person: as a byproduct of having to pay for services.) let the man know that, by law, she has to know his name and some basic contact information before she can provide him with online therapy Rule out this option because having a face-to-face meeting with a client before providing e-therapy is acceptable (and required by some e-therapists), but it is not mandated by any law or ethical standard. Note: the current Ethics Code is not specific with regard to telephone therapy or teleconferencing or any electronically provided services as such and has no rules prohibiting such services. "Psychologists considering such services must review the characteristics of the services, the service delivery method, and the provisions for confidentiality. Psychologists must then consider the relevant ethical standards and other requirements, such as licensure board rules."

A graduate assistant is working on a research study originally initiated by the psychologist. However, the graduate assistant has contributed throughout the duration of the study on defining the research hypothesis, data analysis and developing conclusions based on the results. With respect to authorship, the graduate assistant would be: Select one: A. listed as co-author. B. listed as first author. C. credited in a footnote. D. given no formal credit.

Correct Answer is: A Given the graduate assistant has made a significant contribution to the research study some credit for authorship is due and the graduate assistant would be listed as a co-author. listed as first author. Since the study was the psychologist's original project, it would be appropriate for the psychologist to be listed as the first author. credited in a footnote. The graduate assistant's work extends beyond "Minor or routine professional contributions or extensive nonprofessional assistance (e.g., typing a complicated manuscript, coding data, or helpful ideas offered by a colleague) may be acknowledged in a footnote or in an introductory statement" [See: Keith-Spiegel, P. and Koocher, G. P. (1998). Ethics in psychology: Professional Standards and Cases. New York: Random House]. given no formal credit. The graduate assistant deserves some authorship credit.

You are having lunch with a colleague, Dr. O. Timer, who has been a clinical psychologist for over 25 years. You and Dr. Timer start talking about the aspects of your work that are most distressful. Like many psychologists, Dr. Timer mentions that, most often, the ethical dilemmas he encounters in his practice are related to: Select one: A. deciding when it is acceptable to breach client confidentiality. B. dealing with the sexual advances of his female clients. C. providing accurate information to insurance companies. D. knowing when to continue versus terminate therapy with a client who doesn't seem to be improving.

Correct Answer is: A Surveys have been conducted to determine what issues psychotherapists have the most difficulty with at work. When asked what ethical/legal issue they encounter most frequently, confidentiality is cited most often.

The APA's Record Keeping Guidelines (2007) suggest complete records be retained for: Select one: A. seven years post-termination of therapy. B. 10 years post-termination of therapy. C. three years post-termination and a summary of the record for at least four more years. D. three years post-termination and a summary of the record for at least seven more years.

Correct Answer is: A The APA's Record Keeping Guidelines (2007) indicates a "psychologist strives to be aware of applicable laws and regulations and to retain records for the period required by legal, regulatory, institutional, and ethical requirements." And in the case where there is no legal or institutional regulations, "psychologists may consider retaining full records until 7 years after the last date of service delivery for adults or until 3 years after a minor reaches the age of majority, whichever is later. In some circumstances, the psychologist may wish to keep records for a longer period, weighing the risks associated with obsolete or outdated information, or privacy loss, versus potential benefits associated with preserving the records."

he legal basis of the Tarasoff decision was that Select one: A. the protective privilege ends when the public peril begins. B. psychologists have a legal and ethical responsibility to uphold the general welfare. C. a psychotherapist's duty to enforce the law supercedes his or her duty to protect a patient's confidentiality. D. a psychotherapy patient's right to confidentiality is absolute.

Correct Answer is: A The California Supreme Court's Tarasoff decision established the duty of a psychotherapist to protect the intended victim whenever a patient poses a serious danger of violence to another. In ruling that the need to protect the intended victim supercedes a client's confidentiality rights, the Court wrote that "the protective privilege ends when the public peril begins."

In which of the following circumstances would a psychologist's behavior most likely be considered unethical? Select one: A. hands out her card to individuals in a tent-city whose homes have been destroyed by a hurricane B. receives payment for reviewing a new book in her field, which is published in a popular lay magazine C. has her name and picture featured in a magazine ad in which it is indicated her favorite scotch is Dewars D. sends out announcements to local businesses that she is available to work as an EAP counselor

Correct Answer is: A The Ethical Principles do not ban all forms of solicitation. However, they do ban the solicitation of individuals who are, for whatever reason, "vulnerable to undue influence." Clearly, individuals who have just had their house destroyed are susceptible to undue influence. hands out her card to individuals in a tent-city whose homes have been destroyed by a hurricane Thus, the psychologist in this choice would be acting unethically.

After almost two months, Dr. Roberts realizes the woman he's been dating is the cousin of a client he has been seeing in therapy for six months. The best course of action initially for Dr. Robert's would be to: Select one: A. discuss the matter with the client as soon as possible B. immediately refer the client to another therapist C. continue seeing both unless it is evident that the situation is creating a conflict D. immediately stop seeing the woman

Correct Answer is: A The Ethics Code does not directly address this particular situation however it warns against becoming involved in multiple relationships. Standard 3.05(b) of the Ethics Code states, "If a psychologist finds that, due to unforeseen factors, a potentially harmful multiple relationship has arisen, the psychologist takes reasonable steps to resolve it with due regard for the best interests of the affected person and maximal compliance with the Ethics Code." Consistent with the intent of the Code, the best option would probably be to discuss the matter with the client immediately. While eventually it may be appropriate to refer the client to another therapist or end the relationship with the woman he is dating, neither would be Dr. Robert's the best course of action initially. In other words, the correct choice does not mean that these other choices would not be the final outcome of this situation.

You are working in a clinic as a psychologist and your supervisor, who is working on his Ph.D. in psychology, wants to begin conducting family therapy. Since this is an area which you have expertise in, he asks you to be the consulting supervisor. In this situation: Select one: A. you should refer the supervisor to someone else B. you could only provide the supervision if you first clearly define the roles C. the ethics code does not apply D. you could provide the supervision as long as his clients are informed and consent

Correct Answer is: A The Ethics Code requires that, whenever feasible, psychologists avoid entering into multiple relationships. The best course of action would, therefore, be to refer the supervisor to another qualified professional.

While attending a staff meeting at a mental health facility where you work, you learn that one of your colleagues, a psychologist, is behaving in an unethical manner toward one of her patients. Despite being advised to do so by several members of the staff, she states that she will not change her behavior. What should you do in this situation? Select one: A. Approach the colleague individually and urge her to change her behavior; if she refuses, report her to the appropriate ethics committee. B. Approach the colleague individually and urge her to change her behavior; if she refuses, recommend to the director of the facility that she be fired. C. Report the colleague to the appropriate ethics committee immediately, since she has already stated that she will not change her behavior. D. Announce in the staff meeting that you want to "go on the record" as noting that the psychologist is behaving unethically.

Correct Answer is: A The Ethics Code states that you should deal with ethical violations by another psychologist in an informal manner, by bringing it to the attention of the psychologist. A report to an ethics committee should be made when the attempt at informal resolution has failed or if the violation is not amenable to informal resolution. Thus, you should speak to the psychologist privately and report her to an ethics committee if she still refuses to change her behavior. You may have thought that, since the psychologist has already stated her refusal to behave ethically, you should go ahead and report her, without bothering to talk to her. This actually makes sense, but the Ethics Code requires that you attempt an informal attempt at resolution first. A staff meeting is a formal setting, and does not personally give you the opportunity to attempt an informal resolution.

A psychologist is undertaking a research program in an elementary school on the effect of a new counseling program on enhancing children's self-esteem. The psychologist has received permission from the school's principal, the teachers, and from the parents of each student. However, when she explained the program to the children, some of them objected to being part of the study. The psychologist should Select one: A. not use these children in her study. B. call the parents of these children and ask them to try to convince the children to participate. C. use the children in her study because she has all necessary legal consents. D. re-design the study to eliminate the objection the children have to participating.

Correct Answer is: A The best idea is to just leave out these children. Participants have the right to decline to participate in research, even if they are children. It wouldn't be practical or prudent to ask her to re-design the whole experiment because a handful of children object to being participants. Thus, the best answer is to just do the study without using the children who object to participating.

With parental consent, a psychologist has been providing therapy to a 12-year-old boy for a couple of months. Unexpectedly, both parents suddenly withdraw their consent. The psychologist believes that continuing therapy is in the best interests of the boy. The psychologist's best course of action would be to: Select one: A. seek judicial intervention B. obtain a signed consent from the girl and continue treatment C. stop the treatment because both parents have withdrawn their consent but provide them with referrals to other therapists D. continue to see the girl without the parents' consent

Correct Answer is: A The psychologist's ethical obligation is to serve the best interests of the child, but legally, he/she cannot proceed with treatment if the parents do not give consent to do so. While some states have laws that allow psychologists to provide services without parental consent, no information is given in this question regarding the relevant law or whether the child's problem(s) would be covered by this type of law. The Ethics Code Standard 1.02 states that "If psychologists' ethical responsibilities conflict with law, psychologists make known their commitment to the Ethics Code and take steps to resolve the conflict in a responsible manner." Therefore, the psychologist would not just terminate without attempting to advocate for the patient; nor continue to treat the child without the parents consent, as violating the law is not consistent with resolving the conflict in a responsible manner.

According to APA's Guidelines for Providers of Services to Ethnic, Linguistic, and Culturally Diverse Populations, which of the following suggestions is most useful when working with a client from a minority background? Select one: A. Therapists should treat the client just like any other client. B. Therapists should remember that culture and ethnicity are significantly related to therapeutic issues. C. Therapists should downplay their own culture and emphasize the client's culture. D. Therapists should assume that the client's problems are due to discrimination and bias.

Correct Answer is: B APA's Guidelines for Providers of Service to Ethnic, Linguistic, and Culturally Diverse Populations were designed to provide suggestions to psychologists working with ethnic, linguistic, and culturally diverse populations. One of the suggestions is to "recognize ethnicity and culture as significant parameters in understanding psychological processes."

The director at the training agency, where you are a supervisor, has promised two new interns that they will have plenty of testing experience as part of their training. In order to meet quota, you are under a great deal of pressure to bill many hours. One intern has a great deal of previous testing experience and the other intern has very little. The majority of the testing cases are given to the more experienced intern, and as a result this intern is given the majority of your time and attention. This situation is: Select one: A. ethical B. unethical C. unethical because you are exploiting the more experienced intern D. unethical because the less experienced intern is not getting the training as it was described to him by the director of the clinic

Correct Answer is: A There is no indication that the more experienced intern is being exploited ("unethical because you are exploiting the more experienced intern" ) and while the interns are not receiving an equal number of testing opportunities, the situation is not unethical solely because the less experienced intern is getting fewer testing cases and/or attention. The director stated the interns would have "plenty of testing experience," not necessarily an equal amount of testing cases, time, or attention. Additionally, the definition of plenty is relative to the individual intern, their previous education, training, and experience. Another relative term in the question to consider is "majority." It would be unethical to have the intern with very little testing experience perform more than his/her current level of competency (Standard 2.05(2): Delegation of Work to Others). Note: this intern is receiving test cases, time, and attention - just not the "majority" of cases and attention.

During the course of a research experiment, subjects witness someone who is apparently hurt, needing immediate help, and are unable to help or know if the person is assisted. The subjects become immediately distressed. In this situation: Select one: A. debriefing occurs at the conclusion of participation B. group debriefing occurs at the conclusion of collection of data C. debriefing occurs when subjects become distressed D. debriefing occurs at the conclusion of research

Correct Answer is: A This question involves deception in research, distress and debriefing. Due to the use of deception and the resultant participant distress, debriefing immediately following participation is the best option as described by Standard 8.07(c) Deception in Research: "Psychologists explain any deception that is an integral feature of the design and conduct of an experiment to participants as early as is feasible, preferably at the conclusion of their participation, but no later than at the conclusion of the data collection." The Ethics Code on Debriefing, Standard 8.08(a), notes, "Psychologists provide a prompt opportunity for participants to obtain appropriate information about the nature, results, and conclusions of the research, and they take reasonable steps to correct any misconceptions that participants may have of which the psychologists are aware," and in Standard 8.08(c) "When psychologists become aware that research procedures have harmed a participant, they take reasonable steps to minimize the harm."

Your client has moved out of state. She has called and left you a voice mail message requesting that you send her records to a new therapist. She leaves her new therapist's number and address but not her own. You have a current written release from her on file. You note that she has one outstanding balance of $200. You should Select one: A. contact the therapist and try to get the client's number. B. send all the records. C. send a summary along with an invoice. D. send the summary along with an invoice indicating the amount still owed.

Correct Answer is: A This would be your best choice. It never hurts to take the most conservative approach, which in this case would be to try and contact your client and discuss the situation with her. It will be helpful to know the purpose of the request. Your client has a right to a copy of her records and a right to have them sent to where she chooses, but your best course of action would be to discuss the situation with her. You would not send her bill to another therapist.

Utilization review, an important component of managed health care, refers to the idea that it is useful to Select one: A. review benefits to eliminate or reduce unnecessary health care resources. B. determine the adequacy of health care standards by comparing them to predetermined standards. C. make decisions on patient care by a team of medical experts rather than an individual physician. D. allow a patient to choose from several insurance plans.

Correct Answer is: A Utilization review is concerned with conserving health care monies. It does this through having a utilization review committee assess the use of benefits and reduce or eliminate inappropriate or unnecessary use of health care resources. determine the adequacy of health care standards by comparing them to predetermined standards. This option is a description of the concept of quality assurance. make decisions on patient care by a team of medical experts rather than an individual physician. This option is describing a medical team management approach to individual health care.

A psychologist working for an Employee Assistance Program (EAP) has seen a client who was referred by her supervisor. The supervisor asks the psychologist if the employee has kept the appointment. The psychologist should Select one: A. provide the supervisor only with this information but nothing else. B. refuse to give the supervisor this information. C. provide this information as well as any other information about the client the supervisor is interested in. D. consult with the manager of the company.

Correct Answer is: A When a mental health professional works as part of an EAP, the potential for confidentiality dilemmas is present. Employers, after all, often feel they have the right to know if employees are benefitting from EAPs, but mental health professionals must understand the importance of confidentiality - including, in most circumstances, the confidentiality of whether a person is or is not a client or patient. In the context of EAPs, however, it is considered acceptable to give the relevant information to relevant people; specifically, it's OK to tell the supervisor (or the referring person in the organization) if the employee keeps a scheduled appointment, if treatment is needed, and if the employee has agreed to and accepted treatment. All other information should be kept confidential (barring, of course, the employee's consent to release it).

According to the ethical standards, fees charged to clients are Select one: A. to be established as early as possible in the professional relationship. B. to be established as a function of experience and expertise of the psychologist. C. to be based on community standards. D. are to be as low as practically possible.

Correct Answer is: A to be established as early as possible in the professional relationship. Only this option is stated in the Ethical Principles (see Standard 6.04[a]).

During the 10th psychotherapy session with a client, you realize that your client has recently started dating your best friend. You should: A. refer the client to another therapist B. discuss this with your client immediately C. discontinue your relationship with your friend D. wait and address the problem with your client if a conflict becomes imminent

Correct Answer is: B Although the Ethics Code does not specify exactly how to resolve this type of ethical dilemma, it does state, "If a psychologist finds that, due to unforeseen factors, a potentially harmful multiple relationship has arisen, the psychologist takes reasonable steps to resolve it with due regard for the best interests of the affected person and maximal compliance with the Ethics Code" (Ethical Standard 3.05[b]). Consistent with the intent of the Code, the best option would probably be to discuss the matter with your client immediately. After discussing it with your client, additional steps, such as referring the client, may be appropriate.

At the request of an attorney, a psychologist in a rural community agrees to conduct a psychological evaluation and provide treatment for the same person. This would probably be considered: A. ethical, since both services are within the boundaries of a professional relationship B. ethical, if the psychologist takes reasonable steps to minimize any negative effects C. unethical, since evaluation and treatment are inherently conflicting D. unethical, since only a patient, not an attorney can request treatment services

Correct Answer is: B As a general rule, psychologists should avoid potentially conflicting relationships, which could result when conducting both an evaluation and treatment for a patient. However, according to APA's Specialty Guidelines for Forensic Psychologists (1991), "When it is necessary to provide both evaluation and treatment services to a party in a legal proceeding (as may be the case in small forensic hospital settings or small communities), the forensic psychologist takes reasonable steps to minimize the potential negative effects of these circumstances on the rights of the party, confidentiality, and the process of treatment and evaluation."

Waiving privilege is the right of: A. the courts B. the client C. the client's therapist D. the client's attorney

Correct Answer is: B Privilege is a legal term that refers to a client's right to confidentiality in legal proceedings. The client is the holder of the privilege therefore only the client can waive privilege. Although there are legal exceptions to privilege, the court or the client's attorney cannot waive it. The client's therapist can only "assert" the privilege, not waive it.

A woman seeks help from a psychologist in dealing with her 12-year-old daughter who has been misbehaving lately. The mother says that the daughter has not been listening to her and last week cursed at her mother. The mother reacted by slapping her daughter which left a bruise. The mother feels very remorseful about it and says that she has never done anything like that before. The psychologist should: A. insist that the woman attend anger management classes B. report the incident to a child protective services agency before the end of the session C. monitor the situation and report any further abuse incidents to a child protective services agency D. seek consultation

Correct Answer is: B State laws require mandated reporters to report any reasonable suspicion of child abuse. Abuse generally includes, but is not limited to, physical injury that is inflicted upon a child by other than accidental means. The reporting laws do not provide exceptions for those who are remorseful or first-time offenders. Although the reporting laws do not require the reporter to inform the abuser of the report, or to involve him or her in making the report, it is often clinically useful to do so. Referral to anger management classes might be appropriate, but the psychologist should not insist upon it and the psychologist must still meet the reporting requirements.

A researcher who is interested in determining the correct language to use in referring to members of ethnic minorities in a research article should refer to information published by A. the US Census Bureau. B. the American Psychological Association (APA). C. the research committee in the setting where the research takes place. D. the state licensing board.

Correct Answer is: B The Publication Manual of the American Psychological Association is the authority on questions of style (e.g., word usage, formatting) in research articles. It includes a section on choosing nouns referring to racial and ethnic groups.

You belong to a managed-care panel and a client with a 12-session limit was referred to you. Without calling to cancel, she does not show up for her fourth session. How would you bill for the missed session? A. bill the insurance company B. bill the client according to office policies C. bill the client for the co-payment and the insurance company for the no-show D. you cannot bill either the client or insurance company

Correct Answer is: B The best answer is that you should bill according to your office policies which, presumably, was explained and agreed to by the client at the beginning of treatment (Standard 6.04: Fees and Financial Arrangements). If charging the client for the missed session was not agreed upon in advance, this would be unethical. You cannot bill the insurance company, since you should only bill insurance for services that you actually provide (Standard 6.06: Accuracy in Reports to Payors and Funding Sources). While your financial agreement may or may not require the client to cover the co-payment for missed sessions, Current Procedural Terminology (CPT) codes do not provide a code for no-shows, and billing for services not rendered is unethical.

A defendant's ability to take part in and understand the proceedings against him in a criminal trial is an issue of: A. sanity B. competence C. consent D. testamentary capacity

Correct Answer is: B The situation described in the question refers to competence to stand trial.

The concept of privilege is best described by which of the following statements? A. the primary purpose of privilege is to protect the professional's right to withhold client information B. privilege is a legal term referring to the client's right to not have confidential information released in court without his or her permission C. privilege refers to the ethical obligation of therapists to protect a client's right to privacy D. privilege refers to the responsibility of the professional to safeguard the confidentiality of his or her clients

Correct Answer is: B The term "privilege" refers to the right of a client to prevent confidential information, such as client records or therapist-client communications, from being released in legal proceedings. The client typically holds the right to claim or waive the privilege as "holder of the privilege." However, a psychologist can claim privilege on the client's behalf when his or her testimony or records are subpoenaed.

An ethics committee contacts a psychologist regarding the investigation of an alleged ethics violation by another psychologist. How should the psychologist respond? Select one: A. The psychologist has no ethical responsibility to cooperate with the ethics committee. B. The psychologist should cooperate with an ethics committee. C. The psychologist should wait until ordered by the court before cooperating with the ethics committee. D. The psychologist should obtain the other psychologist's consent before cooperating with the ethics committee.

Correct Answer is: B According to Ethical Standard 1.06 [Cooperating With Ethics Committees], "Psychologists cooperate in ethics investigations, proceedings, and resulting requirements of the APA or any affiliated state psychological association to which they belong. In doing so, they address any confidentiality issues. Failure to cooperate is itself an ethics violation. However, making a request for deferment of adjudication of an ethics complaint pending the outcome of litigation does not alone constitute noncooperation."

An article to be published in a psychology research journal is based on doctoral student's dissertation. The work of the student's thesis advisor, a well-known expert in the dissertation's subject area, was cited extensively in the dissertation and the article. The advisor conducted a follow-up study and added the results of it to the results section of the article. He also modified the conclusion, adding an interpretation of the results of the follow-up study, and a few revisions to the student's wording. In this case, the thesis advisor should be Select one: A. listed as the first author of the article. B. listed as an author of the article. C. should be credited in a footnote. D. should be thanked by the student in text of the article.

Correct Answer is: B According to standard 8.12 of APA's Code of Conduct, "Except under exceptional circumstances, a student is listed as principal author on any multiple-authored article that is substantially based on the student's doctoral dissertation." Nothing in this question describes any exceptional circumstance. Thus, the student should be listed as the first author. However, the advisor substantially contributed to the article by conducting a follow-up study and including the results in the article. This contribution is sufficient to qualify him as one of the authors of the article.

A father brings his 10-year-old son to a psychologist for therapy due to symptoms of depression. The father states that the boy's symptoms began shortly after the parents separated several months ago. The boy is currently living with his father. The father further states that he only wants help for his son and does not want any report or testimony from the psychologist for the divorce proceedings. The psychologist should: Select one: A. have both parents sign an agreement that neither will require the psychologist to provide a report or testimony about the child's therapy B. contact the parents' attorneys to clarify the psychologist's role C. refuse to treat the child unless you can evaluate both parents D. provide therapy for the child but do not maintain any records that could be subpoenaed

Correct Answer is: B Ethical Standard 10.02(b) states, "If it becomes apparent that psychologists may be called on to perform potentially conflicting roles (such as family therapist and then witness for one party in divorce proceedings), psychologists take reasonable steps to clarify and modify, or withdraw from, roles appropriately." Although the patient's father has indicated that he does not want a report or testimony from the psychologist, it would be more prudent to clarify the psychologist's role with the attorneys of each of the parents. have both parents sign an agreement that neither will require the psychologist to provide a report or testimony about the child's therapy Having the parents sign an agreement as suggested in this option does imply a clarification of the psychologist's role, but it may not be legally binding or in the child's best interests and is not as good a choice as the correct option. refuse to treat the child unless you can evaluate both parents This is incorrect because although it is necessary to evaluate all members of a family before making recommendations regarding child custody, in this case, it is therapy that is requested, not a custody evaluation. provide therapy for the child but do not maintain any records that could be subpoenaed This is incorrect because psychologists must maintain adequate records.

You have been seeing a client for several months and believe that treatment has been a success. When you suggest termination to the client, he expresses a desire to continue therapy. Both you and the client are unable to come up with additional treatment goals; however, the client states that he really looks forward to coming to therapy just to talk. In this situation, you should: Select one: A. terminate treatment immediately. B. terminate treatment after a period of pretermination counseling, in which the client's reasons for not wanting to terminate are explored. C. continue seeing the client until he is ready to end therapy. D. seek consultation.

Correct Answer is: B Ethical standard 10.10 requires that a psychologist terminate a professional relationship when it becomes "reasonably clear that the client/patient no longer needs the service, is not likely to benefit, or is being harmed by continued service." Prior to termination, the psychologist must "provide pretermination counseling and suggest alternative service providers as appropriate."

A psychologist is seeing a client who is covered under her husband's insurance policy that he obtained through his employment. As part of the application process for insurance, the husband signed a blanket release of information that permits release of all medical records to the insurer. As a follow-up to a claim the psychologist submitted, the insurer requests all therapy records from the psychologist. The psychologist's best course of action would be to Select one: A. refuse to release any information without contacting the patient first. B. attempt to determine what information is needed to resolve the claim, and only release that information. C. release all of the information to the insurer, but take steps to ensure it is transmitted securely and identifying information related to third parties is redacted. D. release the information without conditions.

Correct Answer is: B Standard 4.04 of APA's Ethical Principles and Code of Conduct applies to this question. It states "Psychologists include in written and oral reports "only information germane to the purpose for which the communication is made." In other words, even though the client's husband has signed a blanket release of information, the psychologist still must take steps to safeguard client privacy. refuse to release any information without contacting the patient first. Discussing the situation with the patient, as stated by this choice, might be appropriate, but the wording of the choice is too absolute--there is no need to "refuse" to disclose "any" information.

The purpose of State Licensing Boards is primarily to Select one: A. educate the public. B. educate the profession. C. protect the public. D. set minimum standards of practice.

Correct Answer is: C The State Boards derive their authority from the principle of protecting the public. That's the justification for giving exams, setting requirements for licensure, writing and enforcing regulations, and so on.

The APA's position on notification of partners of clients/patients with HIV/AIDS is consistent with its emphasis on: Select one: A. duty to protect B. confidentiality C. informed consent D. protection from civil or criminal liability

Correct Answer is: B The American Psychological Association's position on partner notification is consistent with the emphasis on maintaining client confidentiality. Specifically, it recommends "a legal duty to protect third parties from HIV infection should not be imposed." In its statement, Legal Liability Related to confidentiality and the Prevention of HIV Transmission, the APA (1991) also addresses the possibility of such legislation passing and recommends disclosure should be permitted only when (1) there is a known identifiable third party at significant risk for infection; (2) the third party is unaware of the risk; (3) the client has been urged to tell the partner and has refused to do so; and (4) the psychologist is protected legally from civil or criminal liability for disclosing.

he mother of your 8-year-old patient calls to inform you of a meeting being held at the child's school in 30 minutes to discuss the child's recent behavioral problems. The child reportedly has had repeated bouts of yelling and hitting other students. The mother requests that you call the school psychologist to discuss your evaluation of the child's behavior. You should: Select one: A. tell the mother that you're sorry that you can't release any information until she comes in to sign a release. B. agree to provide the information and document the mother's verbal consent to release it in your patient's file. C. call the school and provide them with the minimal amount of information that is appropriate. D. agree to provide the information by phone but follow up with a written report to the school psychologist.

Correct Answer is: B The Ethics Code states that consent to reveal confidential information must be obtained, however, it does not require the consent to be in writing. Documentation of the mother's verbal consent in a progress note could be sufficient, particularly under an urgent situation such as this, in which it is not feasible to obtain a written consent. As suggested in this choice, you should also limit the disclosure to only relevant, though not necessarily "minimal" information.

The purpose of APA's Standards for Educational and Psychological Testing is to Select one: A. provide guidelines for establishing the reliability and validity of tests. B. provide criteria for the evaluation of tests. C. provide guidelines for the use of tests in various settings such as psychotherapy, the workplace, and the courtroom. D. offer psychologists instructions and advice regarding the proper way to administer tests.

Correct Answer is: B The Standards for Educational and Psychological Testing is a set of guidelines published jointly by the American Psychological Association, the American Educational Research Association, and the National Council on Measurement in Education. The most recent version of the Standards, published in 1999, provides 15 sets of guidelines divided into three parts: Test Construction, Evaluation, and Documentation; Fairness in Testing; and Testing Applications. Their purpose, according to the introduction, is to "provide criteria for the evaluation of tests, testing practices, and the effects of test use."

In the last session, your gorgeous new client complimented you on how nice you look in your red outfit. You noticed you were very happy before this session. You even caught yourself singing, "Oh What a Beautiful Morning" while looking in the mirror to do a final 'visual check' before seeing your client in. Ethically, Select one: A. You need to cancel this session B. You need to consult with a colleague C. You need to behave more appropriately D. You need to get out more

Correct Answer is: B While no Ethical Standards have been violated at this point; there is a strong indication that you are experiencing some type of "extra attraction" toward this client. You need to seek consultation and determine how to process and handle your countertransference. Cancelling the session may not be in the client's best interest.

You are a small town psychologist in private practice and a client has written a letter to the Psychology Board complaining that you have a very unprofessional staff. The Board has now written you to inform you of this matter. You should: Select one: A. Discipline your staff B. Respond to the Board C. Call the client and let them know that you will make the appropriate changes with your staff D. Refer the client to a psychologist in another town

Correct Answer is: B You must respond to the Board. To not do so is itself an ethical violation. While it is possible that this is a clinical issue between yourself and your client it is no longer in the therapeutic realm. Your client has gone to an outside source and that source is concerned enough to begin a preliminary investigation.

You are working as a psychologist in a hospital with a patient suffering from head trauma. In your presence, the patient develops Delirium. In this situation, you should: A. bring more people into the room. B. find a physician for a medication consultation. C. stay with the person to provide support and help him remain calm. D. recommend ECT treatment.

Correct Answer is: C "Stay with the person to provide support and help him remain calm" is the best choice for the following reasons: 1) Not all Delirium patients require medication; only if the patient is agitated, psychotic, or can't sleep at night is medication necessary, and 2) "find a physician for a medication consultation" implies that you would leave the patient to go find a physician. You should not leave a patient with Delirium alone, and even if a nurse or somebody else were available to stay with the patient, you should avoid sudden changes in a Delirium patient's environment.

You see an elderly male in therapy. In the first session, he breaks down sobbing and admits that he has recently "helped" his wife, who was suffering from incurable cancer, to die. First you need to A. comfort him as best you can and make a call to the police. B. get the number of a trusted family member and alert them. C. assess your client's potential for suicide. D. suggest that your client call the police while still in your office.

Correct Answer is: C First of all, this question is not about a Tarasoff situation. You are speaking to the husband after the fact of the wife's death so you do not have a mandated breach of confidentiality, and instead now need to decide what is in the best interest of your client. What you have is a bereaved client who is at a high risk for suicide. With his best interests and welfare in mind, you need to assess his potential for suicide ... and then decide if you want to get the number of a trusted family member and alert them.

A psychologist is providing therapy to a client through the client's Employee Assistance Program at work. The client's supervisor can be informed by the psychologist that the client is receiving treatment: A. under no circumstances B. if the client's problem is affecting her job performance C. if the supervisor referred the client to the program D. if that is the only information he reveals to the supervisor

Correct Answer is: C Generally, in the context of an EAP, information about an employee/client's condition or treatment should not be disclosed without the employee/client's consent. However, as the correct answer ("if the supervisor referred the client to the program" ) states, when the employee/client comes to therapy as the result of a referral by his or her supervisor, the supervisor can be given certain limited information: He or she can be told if the employee/client kept the appointment, whether the employee/client needs treatment, and whether the employee/client accepted treatment. The supervisor should not be given any other confidential information unless the psychologist has permission from the employee/client. Confidentiality can also be breached under the same conditions that it can be breached in other contexts (e.g., danger to self, mandated reports).

A psychologist has been treating a client in therapy for four months. The client has not shown up for the last two appointments, and is overdue on payment for an additional two sessions. In this situation, the psychologist's best course of action would be to A. refer the matter to a collection agency. B. terminate treatment. C. contact the client to discuss the situation with him. D. interpret the client's behavior as resistance if and when he shows up for a session.

Correct Answer is: C In this situation, the psychologist must address two issues: the client not showing for two appointments, and the client's failure to pay for two completed sessions. This choice ("refer the matter to a collection agency") does not address the issue of missed sessions, nor is it a really great answer for addressing the payment issue, because it fails to note that a psychologist cannot use a collection agency unless the client was first informed that this could happen and provided an opportunity to pay.

Which of the following statements regarding the need to obtain informed consent before releasing client information is most true? A. A psychologist should always obtain informed consent from the client before releasing information. B. Before consulting with a colleague about a case, a psychologist must always obtain informed consent from the client. C. A psychologist must obtain informed consent from a client some of the time before releasing information. D. If a psychologist wants to release information about a case, he or she may do so without obtaining informed consent.

Correct Answer is: C Ordinarily, you need to obtain informed consent from a client before releasing information you've obtained from him or her. However, this is not always the case; for instance, when you reasonably suspect child abuse, you are legally obligated to file a report with the appropriate state agency -- with or without the client's consent.

To ensure that providers meet and maintain health plan participation requirements, managed care organizations (MCOs) use _____________________ as a review process. A. clinical audit B. concurrent review C. credentialing D. quality management

Correct Answer is: C The formal process for determining if a provider meets and maintains the standards of qualification, as well as providing some legal protection for the managed care organization, is referred to as credentialing. Clinical audit is a quality assurance method. Concurrent review describes the utilization review conducted during the course of treatment. Quality management is another name for quality assurance.

A 14-year old is brought to your office. He does not want to be there. His mother notes that they are referred by the school. The only presenting complaint is that his grades are much poorer than last year. Your evaluation indicates no significant other problematic areas in his life. Your best course of action is to: A. schedule ongoing treatment with yourself. B. continue your assessment to ascertain the presence of hidden pathology. C. explore the possibility of an educational intervention. D. not do anything.

Correct Answer is: C This is a practical, and often realistic, problem. You have a 14-year-old who doesn't want to come to you. You have a referral from the school that his grades are going down. You made an assessment that there doesn't appear, now anyway, to be any psychiatric disorder. So, what would you do? There's no need to suggest treatment since he doesn't want to come see you. And, besides, you haven't found any problems. You wouldn't continue an assessment since, from the information given, you already concluded there isn't much psychological conflict present. You could do nothing, telling him to come back to you if things get worse. That's one possibility. Another possibility presented as a choice is to explore some educational intervention, such as talking to his teachers to investigate how his school environment might be optimized. So you could certainly propose intervening with the school. And between suggesting this, and doing nothing, we'd go with exploring a school-based intervention.

Your managed care company denies to pay for further sessions for a client and requests that you do not tell the client of its decision. In this situation, you should A. terminate with the client as requested. B. continue to provide treatment to the client if you believe the client could benefit from treatment. C. inform the client of all aspects of this decision that are relevant to treatment. D. seek the advice of an attorney regarding a possible lawsuit against the managed care company.

Correct Answer is: C This question is not so much about managed care as it is about informed consent procedures. Whether or not a patient's treatment is being financed by a managed care company, a psychologist should inform clients of relevant information regarding treatment at the outset of the professional relationship and continue to provide such information as it arises throughout treatment. Due to the nature of managed care companies, there may be more concerns that need to be discussed with managed care patients than with other clients. But the general informed consent requirement applies to this situation as well as many others. The other choices don't really address the issue raised by the question.

In cases of litigation, which of the following is considered the key defense for a psychologist? Select one: A. licensure B. liability insurance C. sufficient record-keeping D. informed consent

Correct Answer is: C A review of the risk management literature indicates good record-keeping is the best key defense in cases of malpractice.

You are the only provider in a rural community and your spouse's coworker asks if he could schedule an appointment with you. Which of the following is the best course of action? Select one: A. You must refer him. B. You must schedule an appointment with him. C. You should refer him if you believe that treating him would impair your objectivity. D. You should discuss this with your spouse and this coworker to determine if this constitutes a dual relationship.

Correct Answer is: C According to APA's Ethics Code regarding multiple relationships (3.05[a]), "A psychologist refrains from entering into a multiple relationship if the multiple relationship could reasonably be expected to impair the psychologist's objectivity, competence, or effectiveness in performing his or her functions as a psychologist, or otherwise risks exploitation or harm to the person with whom the professional relationship exists." You must refer him. This choice is too prohibitive. Although it may be necessary to refer this person, if it appears unlikely that entering into a professional relationship would impair your objectivity or would otherwise be harmful to the person, you could treat him. You must schedule an appointment with him. This choice can be eliminated because psychologists and other health professionals have no legal or ethical obligation to enter into a professional relationship. You should discuss this with your spouse and this coworker to determine if this constitutes a dual relationship. This choice can be eliminated because discussing the matter with your spouse would be a breach of confidentiality.

A psychologist can kill an animal in research: Select one: A. as long as the animal is treated humanely B. as long as an anesthetic is used, with an effort to minimize pain C. when it is appropriate and if it is done rapidly, with an effort to minimize pain D. under no circumstances

Correct Answer is: C According to Ethical Standard 8.09(g) (Humane Care and Use of Animals in Research), "When it is appropriate that an animal's life be terminated, psychologists proceed rapidly, with an effort to minimize pain and in accordance with accepted procedures." In all research with animals, the animals must be treated humanely. If surgical procedures are performed, they are done under appropriate anesthesia with techniques to avoid infection and minimize pain.

A psychologist is working for a company that changes health care providers due to cost-effectiveness. The new provider has a policy that compromises the employee's future limits to confidentiality. The psychologist should Select one: A. refuse to follow the new procedures and leave immediately. B. outline her points of dissatisfaction and send it to the new provider. C. make clear with each client the company's new limits to confidentiality. D. organize a strike and call CNN.

Correct Answer is: C According to Standard 3.11 (Psychological Services Delivered To or Through Organizations), "Psychologists delivering services to or through organizations provide information beforehand to clients and when appropriate those directly affected by the services about (1) the nature and objectives of the services, (2) the intended recipients, (3) which of the individuals are clients, (4) the relationship the psychologist will have with each person and the organization, (5) the probable uses of services provided and information obtained, (6) who will have access to the information, and (7) limits of confidentiality. As soon as feasible, they provide information about the results and conclusions of such services to appropriate persons."

According to the most recent version of APA's Ethical Standards and Code of Conduct, bartering goods and services in exchange for psychotherapy Select one: A. is prohibited. B. should be refrained from under ordinary circumstances. C. is acceptable if it is not clinically contraindicated and not exploitative. D. is acceptable if the practice is common in the area where the therapist is practicing.

Correct Answer is: C According to standard 6.05 of APA's Ethical Standards and Code of Conduct (2002), "Psychologists may barter only if (1) it is not clinically contraindicated, and (2) the resulting arrangement is not exploitative." The previous version of the standards contained a statement to the effect that psychologists ordinarily refrain from bartering, but that statement was removed in the most recent version, perhaps in recognition of the fact that bartering is a common arrangement in some communities and may be the only option for cash poor clients. Nonetheless, psychologists should proceed cautiously before entering into a bartering arrangement. Issues to consider include whether a fair value for the bartered goods/services can be established, and clinical considerations such as whether the nature of the transference would interfere in arranging a non-exploitative bartering relationship. Exchanging therapy for clerical, housekeeping, or other ongoing services creates a potentially harmful multiple relationship that would violate this Standard (e.g., because, if the client does a poor job, the therapist's objectivity may be impaired). However, barter may be acceptable when a client cannot afford to pay for therapy and it is possible to establish an equitable exchange. It would likely be acceptable, for example, to establish a barter arrangement with a farmer who can't afford to pay for therapy and offers produce in exchange for therapy as long as the value of the produce can be established

You are seeing a 15 year old girl in therapy. Prior to beginning treatment, you, the girl, and her parents agree that you will maintain the girl's confidentiality and not discuss anything she reveals in therapy with her parents. In the first session you realize that the girl is very depressed, and during the fifth session she tells you that she wants to kill herself. Under these circumstances, you will: Select one: A. not contact her parents but have the girl sign a "no-suicide contract." B. contact her parents only if they are unaware of the girl's depressed state. C. contact her parents if you feel the girl is at high risk for attempting suicide. D. contact the child protective agency.

Correct Answer is: C Even though you've come to an agreement with the parents and the adolescent, the parents are still the holder of the privilege. Also, whenever someone is a threat to themselves, you have an obligation to take steps to protect them, even when this means violating confidentiality.

In cases of ethical violations investigated by the APA following the loss of a psychologist's license, which of the following is the most common ethical issue involved? Select one: A. multiple relationships (nonsexual) B. practice outside the boundaries of competence C. sexual misconduct D. confidentiality breaches

Correct Answer is: C Every year, the Ethics Committee files a report on the number and types of ethical complaints against psychologists it has investigated. In cases in which the committee investigated a complaint following a state board's revocation of a psychologist's license, sexual misconduct has been the ethical issue involved in the preponderance of cases.

From a legal standpoint, a charge of malpractice against a psychologist would be held valid if it were proven that the therapist: Select one: A. had a duty of care to the patient, held malevolent intentions toward the patient, and engaged in activity that resulted in harm to the patient. B. had a duty of care to the patient and held malevolent intentions toward the patient; demonstration of actual harm is not necessary. C. had a duty of care to the patient and engaged in activity that resulted in harm to the patient; malevolent intentions need not be present. D. engages in activity that harms a patient; malevolent intentions and a duty of care are not necessary.

Correct Answer is: C For a malpractice claim against a psychologist to be held valid, three elements must be proven: 1) the psychologist must have had a professional relationship with (and, therefore a duty of care to) the client; 2) the psychologist must have been negligent or failed to live up to that duty; and 3) harm to the patient must have resulted. In other words, the psychologist's intentions are not an issue.

A quality assurance review would likely focus on all of the following except Select one: A. treatment effectiveness. B. access to treatment. C. cost of treatment. D. patient satisfaction.

Correct Answer is: C In the health care business, quality assurance reviews are conducted by organizations such as HMOs or independent overseers to review the quality of services provided. Because the cost of services is not directly related to their quality, it would probably not be a focus of a quality assurance review.

A psychologist discovers a client is HIV positive, has not informed his/her partner and is having unsafe sex with the partner. Current laws related to the psychologist's obligation is most accurately described by which of the following? Select one: A. Health professionals are legally required to breach confidentiality in this situation. B. Health professionals are legally required to NOT breach confidentiality in this situation. C. Health professionals are legally required to breach confidentiality in this situation in some states and prohibited in others. D. Health professionals are legally allowed to determine whether or not to breach confidentiality in this situation.

Correct Answer is: C In this situation, the laws regarding breach of confidentiality vary from state to state and are inconsistent. Some states have laws that prohibit practitioners from notifying the partners of HIV/AIDS clients, and some state laws require practitioners to inform the partners.

Your new patient is a 20-year-old gang member who informs you that he has killed a couple of people in the past and that he will probably kill others in the future. He states that he has no intentions of leaving the gang but wants help dealing with his cocaine addiction. You should: Select one: A. notify the police of the patient's previous murders B. notify the police of the patient's danger to others C. provide the patient with substance abuse treatment D. refuse to provide treatment for this patient unless he agrees to leave the gang

Correct Answer is: C Many states have laws based on Tarasoff that require psychologists to warn or protect a potential victim from serious harm when there is an imminent threat to an identifiable victim. However, this question does not indicate an imminent threat nor an identifiable victim; therefore, confidentiality should be maintained. The Ethics Code similarly permits a breach of confidentiality without client consent in order to protect others from harm, but without an identifiable victim such a breach would not serve to protect others and would be inappropriate. The best option would be to provide the treatment requested. Refusing to provide treatment for this patient unless he leaves the gang would be a clinical option, but would probably be ineffective at this stage because he has stated that he has no intentions of leaving the gang. Besides, engaging the patient in treatment for his substance abuse might later lead him to want to leave the gang.

You have been seeing a female client in therapy for over a year and on your birthday she arrives with a beautifully gift-wrapped sweater and birthday card. You should Select one: A. explain that it is unethical and illegal to accept anything from a client. B. keep the card, but don't accept the sweater. C. explore the meaning of the gift. D. agree to keep the gift if the client will exchange it for a different color ""you have already received quite a few green sweaters.

Correct Answer is: C None of the answers are quite right, but this response ("explore the meaning of the gift") is the best choice. It suggests you first try to explore the clinical transference. However, once investigated, the resolution probably won't include a new sweater. explain that it is unethical and illegal to accept anything from a client. This choice is incorrect because while it is usually unethical to accept gifts, it is not illegal to do so. keep the card, but don't accept the sweater. This is a simple solution, but doesn't demonstrate any clinical exploration of the situation.

A new client comes to see you and you find his political views to be distasteful. In this situation, as an ethical psychologist, you should: Select one: A. refer the client to someone else. B. discuss your feelings with the client and assess to determine if his views are reflective of underlying psychopathology. C. respect the client's political views and proceed with therapy. D. seek consultation.

Correct Answer is: C Principle E (Respect for People's Rights and Dignity) states that "Psychologists are aware of and respect cultural, individual, and role differences, including those based on age, gender, gender identity, race, ethnicity, culture, national origin, religion, sexual orientation, disability, language, and socioeconomic status and consider these factors when working with members of such groups." respect the client's political views and proceed with therapy. Therefore, this is the best answer, since it is most consistent with this ethical principle. A consultation or even a referral may be necessary if your ability to conduct therapy would be impaired. However, there is no reason why a difference of political opinion alone should interfere with therapy.

The police inform you that they have a warrant out on one of your clients who is suspected of several felony charges of grand theft auto. They want you to provide them with the client's address and phone number. You should: Select one: A. cooperate with the police B. require them to get a warrant before you release any information C. refuse to provide them with the information and notify your client D. inform them when your next scheduled appointment is with your client

Correct Answer is: C Since there is no indication that your client is in imminent danger to self or others, you would be obligated to protect the confidentiality rights of your client. However, you should notify your client of this information that you received, and discuss the option of turning himself in to the police.

A psychologist is asked by the attorney for the father in a divorce procedure to determine, in his expert opinion, which parent is better able to care for the children. The father is in the same state as the psychologist but the mother is in a distant state. The psychologist should Select one: A. agree to evaluate the father and give his opinion as requested as long as the conclusions are not influenced by who pays the fee. B. evaluate the father but not send the report until asked for it by the judge. C. evaluate the children and both parents before giving such an opinion. D. refuse to accept this work since there can be no firm predictions made as to which parent would be better able to care for the children.

Correct Answer is: C The basic principle is that you can't give an opinion as to which parent will be better able to care for the children unless you interview both parents and the children. Otherwise, all you can attest to is your findings about the psychological health of whomever you evaluate. You can't, if you only see one of the parents, opine as to which one is better: You'd have no basis to come to such a conclusion.

The friend of a psychologist is the owner of a small publishing firm. The friend offers the psychologist substantial compensation if she would lend her name to endorsements for his publications, and at times advise and consult on book acquisitions and marketing plans. The psychologist: Select one: A. could accept this as long as APA affiliation is not mentioned in the endorsements. B. could accept this only if the publications she would be dealing with are in her area of competency. C. could accept this only if the publications she would be dealing with are in her area of competency, and the endorsements will be based on the psychologist's honest opinion of the publications. D. should consult the local ethics committee before making her decision.

Correct Answer is: C The ethical standards say that psychologists cannot make false, deceptive, fraudulent, or misleading statements. Unless the psychologist's endorsements were based on sound expertise and her honest opinion of the publications in question, these standards would be violated.

Therapists who become sexually involved with their clients Select one: A. usually do so because they believe they are in love with the client as a result of the client's unique and desirable characteristics. B. usually do so as a result of a lack of experience and poor judgment. C. usually do so to fulfill personal needs resulting from events in their own life. D. do not show any characteristics that distinguish them from therapists who do not become sexually involved with their clients.

Correct Answer is: C The experts generally agree that therapists who have sexual relationships with their clients are not only exhibiting poor judgment but are also usually suffering from some type of impairment, often related to boundary issues or unfulfilled needs.

You have been asked by your client's attorney to serve as a fact witness in a court proceeding. Your client, the plaintiff, suffered a serious physical injury in the workplace and has since experienced significant emotional problems as a result of the injury. You have been working with the client for the past several months. During your testimony, you are asked by the plaintiff's attorney if, in your opinion, the client's emotional problems are directly related to the injury. You should: Select one: A. provide the information as long as you have sufficient data to support your opinion. B. provide the information only if the plaintiff has waived her right to confidentiality. C. not provide the information unless ordered to do so by the court. D. provide the information, but note any possible limitations to your opinion.

Correct Answer is: C The key term in this question is that you are a "fact witness". Unlike an expert witness who can offer opinions, a fact witness, as its name implies, can only testify about facts. A psychologist, while serving as a fact witness, could give an opinion, but only if ordered by the court and any opinions should be qualified by a statement about its limitations.

If a client makes a credible threat of violence, the least acceptable course of action would be to Select one: A. initiate involuntary hospitalization procedures in accordance with your state or province's law. B. warn the intended victim. C. provide a safe place for the client to express his anger. D. inform the police.

Correct Answer is: C This question relates to what is commonly called the "Tarasoff rule," or the duty to protect the victim when a client threatens violence against an identifiable victim. In two rulings in the case of Tarasoff v. Regents of the University of California (1974 and 1976), The California Supreme Court ruled that, when a psychotherapy patient makes a specific threat of violence against a reasonably identifiable victim or victims, the therapist must make reasonable efforts to protect the victim(s). Though Tarasoff was a California case, many other states have adopted laws with substantially similar language. The duty to protect is a legal mandate in most states, and in states where it isn't mandated, there are laws the permit psychotherapists to breach confidentiality in order to protect the victim. Thus, the duty to protect has evolved into a standard of care in the field of psychotherapy, and APA's ethics code specifically notes the breaches of confidentiality are permitted "to protect others from harm." Therefore, when a client makes a credible threat of imminent violence against an identifiable victim, a psychologist would need to find an appropriate intervention that meets the duty to protect. provide a safe place for the client to express his anger. Of the choices listed, all but this option are reasonable steps one could take to protect the victim from harm; they perhaps would be performed in combination. This choice seems least likely to protect the safety of the intended victim, and while it may help the client, it fails to address the issue raised by the question.

Dr. Que has recently started working with a family consisting of a mother, father, and five year old boy. The parents say the boy has become "impossible to manage" and they don't know what to do to control him. Dr. Que believes he will get a better idea of the family's dynamics if he is able to observe them at home. When he suggests this, the parents say that, because of their religious beliefs, they cannot allow this. Dr. Que should: Select one: A. refer the family to a therapist from the same religious background. B. impress upon the parents his need to observe them at home in order to best serve the interests of the child. C. determine if there is an alternative and mutually agreeable location for the observation. D. contact child protective services immediately.

Correct Answer is: C This type of situation is covered by ethical requirements related to cross-cultural counseling. There's certainly not any information given that warrants a call to child protective services. Also, it's not necessary to make a referral, and it wouldn't be a good idea to manipulate the parents into doing something they don't want to do.

A psychologist learns from a friend that another psychologist has been revealing confidential information about his patients. Both psychologists live in a small rural community and the information the offending psychologist revealed has apparently spread rapidly around town. The psychologist who learned of this behavior should Select one: A. call the other psychologist's clients to assess the degree of harm that their therapist's behavior caused. B. seek consultation. C. report an ethics violation to a state or national Ethics committee. D. do nothing.

Correct Answer is: C You might have been looking for an answer such as "try to resolve this issue informally by speaking to the other psychologist" , but no such choice was present. Thus, you had to choose the best answer, and as with all ethics questions, the best answer is the one most consistent with the language of APA's Ethical Standards. The following excerpts from Ethical Standards 1.04 and 1.05 apply here:"When psychologists believe that there may have been an ethical violation by another psychologist, they attempt to resolve the issue by bringing it to the attention of that individual, if an informal resolution appears appropriate and the intervention does not violate any confidentiality rights that may be involved." "If an apparent ethical violation has substantially harmed or is likely to substantially harm a person or organization and is not appropriate for informal resolution ... or is not resolved properly in that fashion, psychologists take further action appropriate to the situation. Such action might include referral to state or national committees on professional ethics, to state licensing boards, or to the appropriate institutional authorities." In a nutshell, these standards say that, when another psychologist commits an ethical violation, your responsibility (assuming no confidentiality rights are violated) is to either bring the matter to the other psychologist's attention, or take action such as making a report to an ethics committee. Since this question does not include the option of addressing the issue informally, the best answer must be to report the matter to an Ethics committee.

former patient and yourself meet at the shopping mall 26 months after she has completed therapy with you. Which is the most unethical choice? Select one: A. You help her choose her fall wardrobe. B. She has started smoking again, so you refer her to a smoking cessation group. C. You pursue a romantic relationship with her. D. She is a physician; describe your current physical symptoms and ask her opinion.

Correct Answer is: C You would never PURSUE a romantic relationship with a former client if you are an ethical psychologist. Standard 10.08 (Sexual Intimacies With Former Therapy Clients/Patients) states, "(a) Psychologists do not engage in sexual intimacies with former clients/patients for at least two years after cessation or termination of therapy. (b) Psychologists do not engage in sexual intimacies with former clients/patients even after a two-year interval except in the most unusual circumstances. Psychologists who engage in such activity after the two years following cessation or termination of therapy and of having no sexual contact with the former client/patient bear the burden of demonstrating that there has been no exploitation, in light of all relevant factors, including (1) the amount of time that has passed since therapy terminated; (2) the nature, duration, and intensity of the therapy; (3) the circumstances of termination; (4) the client's/patient's personal history; (5) the client's/patient's current mental status; (6) the likelihood of adverse impact on the client/patient; and (7) any statements or actions made by the therapist during the course of therapy suggesting or inviting the possibility of a posttermination sexual or romantic relationship with the client/patient."

A psychologist's new patient decides to bring in her 12 year-old daughter to deal with communication and relationship issues after a few sessions. The psychologist is confident in being able to help the mom and the daughter get along better. The patient's insurance won't cover family sessions so the psychologist bills the insurance company as if the mom is coming in alone for Generalized Anxiety Disorder. This is an example of: A. therapeutic drift B. unconscious fiscal convenience C. countertransference D. insurance fraud

Correct Answer is: D Despite the psychologist wanting to assist the patient in getting covered services, any form of collusion or lying to an insurance company is insurance fraud. Therapeutic drift refers to the deviation from the protocol and gradual alteration of treatment plan, shifting style & approach for a given client. Unconscious fiscal convenience refers to when a psychologist, unconsciously, overlooks important therapeutic dimensions of sessions because they would conflict with the psychologist's financial self-interest.

A manufacturing company hires a psychologist to screen job applicants using standardized cognitive ability tests. The company then asks the psychologist to train their human resources staff to administer and score the tests. The psychologist should: A. agree to do so if the psychologist is able to provide adequate training to the staff B. agree to do so if the psychologist is able to supervise the human resources staff C. refuse to do so because cognitive ability tests are not valid predictors of job performance D. refuse to do so because the human resources staff lacks the appropriate qualifications

Correct Answer is: D Ethical Standard 9.07 states, "Psychologists do not promote the use of psychological assessment techniques by unqualified persons." A human resources staff would be unqualified to use these tests and should not be trained in their use. Also with regards to, "refuse to do so because cognitive ability tests are not valid predictors of job performance"-- cognitive ability tests are considered to be a relatively good predictor of job performance.

Which of the following statements regarding informed consent procedures in research is least true? A. Psychologists should inform research participants, in language that is reasonably understandable, about the nature of the research. B. Research participants should be informed of significant factors that may affect their willingness to participate in the research. C. Informed consent to research must be appropriately documented. D. Informed consent to research must be obtained in all cases.

Correct Answer is: D In most cases, informed consent to research must be obtained from participants. However, there are some exceptions, such as research involving anonymous questionnaires or naturalistic observations. Ethical standard 8.05 discusses the circumstances under which research does not require informed consent.

"Pro bono" services are ________________ by the APA's Ethics Code. A. not addressed B. prohibited C. required D. encouraged

Correct Answer is: D Pro bono services are professional services that are provided at no charge for the public good. While not explicitly mentioned in the Ethics Code, pro bono services are addressed in the Code's aspirational General Principle B, "psychologists strive to contribute a portion of their professional time for little or no compensation or personal advantage." Since they are addressed in the Code's aspirational General Principles instead of its mandatory Ethical Standards, pro bono services are recommended - but not required - by the Ethics Code.

You have been attempting to collect payment from a former patient of yours who did not pay for his final two months of therapy. One day, you get a letter from a state hospital where that former patient is now currently a patient. The letter includes a request for a copy of your records of the patient's treatment along with a signed release of information. In this situation, you should A. refuse to supply any information to the hospital until the bill is paid. B. send a summary of the records to the hospital along with a copy of a bill for the unpaid therapy fees. C. go to the hospital and talk to the patient about the situation. D. provide the information requested by the hospital.

Correct Answer is: D The issue here is that you can't withhold records under your control that are "requested and needed for a client's/patient's emergency treatment solely because payment has not been received (Ethical Standard 6.03). Thus, you must provide the requested information in this instance.

A psychologist working in the prison system is asked to evaluate a prisoner and make a recommendation about his eligibility for parole. The psychologist should A. refuse to do so since this is outside the scope of a psychologist's work. B. do the evaluation and report its results but refuse to make a specific recommendation. C. make a recommendation as long as it is based on the results of valid standardized tests. D. make a recommendation but point out any possible limitations of the data on which it is based.

Correct Answer is: D This answer is most consistent with the requirements of the Ethics Code and the Specialty Guidelines for Forensic Psychologists. For example, Standard 9.06 of the Code requires that psychologists "indicate any significant limitations of their interpretations."

A psychologist sends the MMPI to a client through the mail. The client will complete the test at home and return it to the psychologist by mail. This is: A. acceptable as long as the client has signed a statement saying she will not breach test security. B. acceptable as long as the psychologist provides adequate instructions over the phone. C. acceptable as long as the test is sent by certified mail. D. unacceptable.

Correct Answer is: D This is pretty clear: As a psychologist, you're responsible for maintaining test security. In addition, you'd want to administer the MMPI under controlled conditions to obtain valid results. If you sent the test to the client, you'd have no control over testing conditions.

Prior to conducting a court-ordered evaluation of a minor, you: Select one: A. must obtain informed consent from the minor. B. must obtain informed consent from the parents or legal guardians of the minor. C. must insure that the parents or legal guardians are present during the evaluation. D. should discuss the purpose of the evaluation with the minor.

Correct Answer is: D APA's "Specialty Guidelines for Forensic Psychologists" states that "psychologists obtain informed consent unless the service is court ordered." Thus, regardless of whether the examinee is an adult or a minor, in a court-ordered evaluation, it is not necessary to obtain informed consent. However, the subject of an evaluation should always be informed of the purpose, methods, and intended use of the evaluation.

Which of the following is most appropriate for referring to a group of research participants? Select one: A. borderlines B. disabled persons C. individuals confined to a wheelchair D. people who have cerebral palsy

Correct Answer is: D According to APA's Publication Manual (2001), authors should avoid perpetuating demeaning attitudes in their writing. It is recommended that people be put before their disabilities and not labeled as their disabilities. Thus, "borderlines" would more accurately and sensitively be referred to as "people diagnosed with borderline personality disorder." Similarly "disabled persons" are better described as "people with (or who have) a disability." The phrase, "individuals confined to a wheelchair" does put the people first; however, it is overly negative and suggests continued helplessness, which should be avoided. Instead, the manual recommends using emotionally neutral expressions, for example, "individuals who use a wheelchair."

Dr. Bean is a newly licensed clinical psychologist who is practicing in a small rural town. She is the only mental health practitioner in a 150-mile radius. Dr. Bean finds that some of her clients have mental health problems that she has little or no experience treating. Her best course of action is to Select one: A. refuse to treat the clients who have problems that she is unfamiliar with. B. obtain appropriate training before treating the clients who have problems that she is unfamiliar with. C. inform the clients of her lack of experience and let them decide what to do. D. see the clients but obtain supervision or consultation by telephone.

Correct Answer is: D According to Ethics Standard 2.01(d), "When psychologists are asked to provide services to individuals for whom appropriate mental health services are not available and for which psychologists have not obtained the competence necessary, psychologists with closely related prior training or experience may provide such services in order to ensure that services are not denied if they make a reasonable effort to obtain the competence required by using relevant research, training, consultation, or study." Since Dr. Bean is the only mental health professional in town, this is the best course of action of those given in the responses since there is no indication that the clients have problems that Dr. Bean doesn't want to treat or that she would not be able to treat with consultation or supervision.

A psychologist is informed by a new client at the conclusion of an initial therapy session that he/she is also seeing a psychiatrist. In this case, the psychologist should: Select one: A. inform the client it would be unethical to continue seeing him/her while under the care of another professional. B. discuss with the client when to terminate treatment with the psychiatrist. C. contact the psychiatrist to inform her of the situation and obtain copies of the client's records. D. discuss the treatment issues and role of the psychiatrist with the client.

Correct Answer is: D According to Standard 10.04 of the Ethics Code, "In deciding whether to offer or provide services to those already receiving mental health services elsewhere, psychologists carefully consider the treatment issues and the potential client's/patient's welfare. Psychologists discuss these issues with the client/patient or another legally authorized person on behalf of the client/patient in order to minimize the risk of confusion and conflict, consult with the other service providers when appropriate, and proceed with caution and sensitivity to the therapeutic issues." The Ethics Code does not prohibit the psychologist in this situation from continuing to treat the client* and contacting the psychiatrist* would violate the client's confidentiality unless it is done with her consent, which is not indicated (* incorrect responses).

A forensic psychologist is asked to review and make conclusions regarding a defendant's psychological evaluation that was conducted 4 years ago following a similar crime that the defendant is now accused of. The psychologist should: Select one: A. refuse the request B. comply with the request but interpret the previous findings with caution C. reevaluate the person and disregard the previous findings D. reevaluate the person and interpret the previous findings with caution

Correct Answer is: D According to the Forensic Specialty Guidelines, "Forensic psychologists avoid giving written or oral evidence about the psychological characteristics of particular individuals when they have not had an opportunity to conduct an examination of the individual..." And, according to Ethical Standard 9.08, psychologists also do not base their decisions on outdated results. Thus, the psychologist in this case should make a reasonable effort to reevaluate the person. However, it would probably be inappropriate to completely disregard the previous findings. The best approach would be to reevaluate the person and interpret the previous findings with caution.

If you receive a subpoena to appear in court regarding one of your clients, you should: Select one: A. Recopy all of your notes so they are typed and not handwritten B. Eliminate all referral data from the file C. Eliminate anything from the file regarding anyone other than your client D. Organize the data and discuss it with your client

Correct Answer is: D After receiving a subpoena you should always discuss with your client the implications of providing the requested information and seek their consent. It is certainly appropriate to organize the data, however, it would not be appropriate, nor ethical or legal, to tamper with the records in any way.

According to the Specialty Guidelines for Forensic Psychologists (APA, 1991), if a psychological evaluation is ordered by a court and the defendant to be evaluated informs the evaluating psychologist that he does not wish to cooperate, the psychologist should: Select one: A. conduct the evaluation B. refuse to conduct the evaluation C. refuse to conduct the evaluation unless the defendant consents D. postpone the evaluation and notify the defendant's attorney

Correct Answer is: D Although it is not necessary to obtain informed consent for court-ordered evaluations, the subject of the evaluation should be informed of its purpose. Furthermore, if the subject does not want to be evaluated, the subject's attorney should be contacted before taking further action. If the attorney also objects to the evaluation, the psychologist should then notify the court and respond as directed.

A psychologist is hired by the defendant's attorney to serve as an expert witness in a criminal case. The arrangement includes a bonus fee for the psychologist if the client is found not guilty. This arrangement is Select one: A. ethical. B. ethical as long as it is disclosed to the court. C. ethical as long as the fee is not excessive. D. unethical.

Correct Answer is: D An expert witness by definition is supposed to clarify issues of fact in a trial and is not supposed to act as an advocate for one side or the other, even if he is hired by one side. For this reason, APA's Specialty Guidelines for Forensic Psychologist explicitly state that psychologists do not provide services to parties in a legal proceeding on a contingent fee basis when those services involve expert testimony. Although the Specialty Guidelines are aspirational rather than binding, APA's Ethical Principles and Code of Conduct, which is binding, contains standards which clearly apply to this situation, such as the prohibition on assuming roles that involve a conflict of interest.

A psychologist determines clients' fees based on their current income. This "sliding scale" practice is: Select one: A. explicitly prohibited in the Ethics Code B. explicitly recommended in the Ethics Code C. unacceptable but not explicitly mentioned in the Ethics Code. D. acceptable but not explicitly mentioned in the Ethics Code.

Correct Answer is: D As long as they are fair and serve the best interests of the client, sliding scale fees are generally considered acceptable. The Ethics Code does not explicitly address sliding scale fees.

A managed care company asks a psychologist to conduct a study on the relative effectiveness of a 10-week therapy program for depression vs. a 20-week therapy program. The company plans to publish the results of the research if no significant difference is found but will not publish the results if the longer therapy program is more effective. The psychologist should: Select one: A. agree to conduct the study because it is within the company's rights to publish or not publish their research findings B. agree to conduct the study if the psychologist is able to approve any article submitted for publication C. agree to conduct the study only if all research participants are appropriately debriefed at the conclusion of the study D. refuse to conduct the study

Correct Answer is: D Ethical Standard 5.01 states, "Psychologists do not knowingly make public statements that are false, deceptive, or fraudulent, concerning their research, practice, or other work activities or those of persons or organizations with which they are affiliated." Although in this case the psychologist is asked not to publish data, this would still be deceptive. It also is inconsistent with General Principle A of the Ethics Code which states that, "psychologists seek to safeguard the welfare and rights of those with whom they interact professionally and other affected persons.... Because psychologists' scientific and professional judgments and actions may affect the lives of others, they are alert to and guard against personal, financial, social, organizational, or political factors that might lead to misuse of their influence."

An experimental treatment that has been used successfully with animals only has recently been approved for research with human participants. Which of the following statements regarding the need to inform participants of the experimental status of the treatment is true? Select one: A. There is no need to inform participants that the treatment is experimental, since the treatment was shown to be safe in animal research. B. There is no need to inform participants that the treatment is experimental, because they would not likely understand the connotations of the word "experimental." C. There is no need to inform participants that the treatment is experimental as long as the study was approved by an institutional review board. D. Participants should be informed that the treatment is experimental.

Correct Answer is: D Ethical Standard 8.02(b) states that "Psychologists conducting intervention research involving the use of experimental treatments clarify to participants at the outset of the research (1) the experimental nature of the treatment; (2) the services that will or will not be available to the control group(s) if appropriate; (3) the means by which assignment to treatment and control groups will be made; (4) available treatment alternatives if an individual does not wish to participate in the research or wishes to withdraw once a study has begun; and (5) compensation for or monetary costs of participating, including if appropriate, whether reimbursement from the participant or a third-party payor will be sought."

You have been treating a client for six months, and the original problem the client sought treatment for is in remission. The client tells you that he wants to terminate. You, however, are reluctant to terminate because you believe that the client can benefit from further psychotherapy. In this situation, you should: Select one: A. terminate in line with the client's wishes, since the client's presenting problem is solved. B. seek consultation. C. discuss the client's reasons for wanting to terminate. D. discuss the reasons why you feel he shouldn't terminate, and, if he still wants to terminate, allow him to do so.

Correct Answer is: D Even though you might not have too much difficulty handling this situation in real life, this question might have been difficult because none of the choices is clearly "wrong." discuss the reasons why you feel he shouldn't terminate, and, if he still wants to terminate, allow him to do so. This is the best answer -- it's in the client's best interests that you explain why you feel he should not terminate, but, if he chooses to do so, you must respect his right to self-determination. terminate in line with the client's wishes, since the client's presenting problem is solved. This choice does not include your explaining your concerns to the client, so it is not as good. seek consultation. You might have been seduced by this option, because seeking consultation is never a bad thing to do, but it is not necessary in this common and relatively straightforward situation. discuss the client's reasons for wanting to terminate. Similarly, when clients want to terminate, it's always important to discuss their reasons, as stated by this choice. In this case, however, the client's reasons are pretty clear -- the problem for which he sought therapy is solved.

Which of the following is a mental health service that HMOs must provide under federal requirements? Select one: A. pre- and post-pregnancy counseling. B. psychodynamically oriented psychotherapy. C. inpatient psychiatric care. D. short-term outpatient evaluative crisis intervention.

Correct Answer is: D Federal laws regarding qualification for an HMO requires that the company provide "short-term (not to exceed 20 visits), outpatient evaluative crisis intervention services", and "medical treatment and referral for alcohol and drug addiction." These are the only qualifications related to mental health services. Most HMOs provide certain additional services, because doing so reduces their costs in the long-run.

A woman comes to you who has been put on morphine due to her pain. She is an ex-alcoholic in recovery and does not want to become addicted to the morphine. She asks your assistance in lowering the dosage. You should: Select one: A. Work with her physician to lower the dose but explain that synthetic morphine isn't addicting B. Tell her she needs to speak directly to her physician C. Suggest she give you the extra morphine and you will dispose of it properly D. Support her choice to not get addicted and work with her physician

Correct Answer is: D It is really alright to support your client's idea even if it isn't yours. You would want to get a signed release to speak to the prescribing physician and work with him or her to support your client's choice.

Which of the following statements regarding therapist-client sexual relations is, according to research, most correct? Select one: A. Therapists treating clients with a diagnosis of Borderline Personality Disorder, Histrionic Personality Disorder, or who have a history of incest are more likely to become sexually involved with their patients than other therapists. B. Therapists who become sexually involved with their patients are likely to have less formal education or to have received less professional recognition than other therapists. C. Therapists who have completed a personal course of therapy are less likely to become sexually involved with their patients than therapists who have not undergone therapy. D. Therapists who become sexually involved with a patient typically do so with a patient who is significantly younger.

Correct Answer is: D Kenneth Pope, in an appendix of his book Sexual Feelings in Psychotherapy (Washington, DC: APA, 1993), provides a review of the research in the area of client-therapist sexual relations. Only the correct choice reflects research findings -- one study found that the average ages of therapists and clients sexually involved with each other were 43 and 33, respectively; another study found these ages to be 42 and 30. Therapists treating clients with a diagnosis of Borderline Personality Disorder, Histrionic Personality Disorder, or who have a history of incest are more likely to become sexually involved with their patients than other therapists. In contrast to this option, no research supports the notion that a particular diagnosis or history places a patient at greater risk for sexual involvement with a therapist. Therapists who become sexually involved with their patients are likely to have less formal education or to have received less professional recognition than other therapists. This is also incorrect -- in fact, one study found that psychologists who had attained a high level of professional achievement reported a higher rate of sex with patients. Therapists who have completed a personal course of therapy are less likely to become sexually involved with their patients than therapists who have not undergone therapy. Similarly, in regard to this choice, one study found that psychologists who had completed psychotherapy or psychoanalysis were more likely to have become sexually involved with a patient than those therapists who hadn't undergone therapy. According to Pope, the best single predictor of sexual exploitation in therapy is a therapist who has exploited a patient in the past.

Dr. Emm believes that one of his patients would benefit from in-patient treatment. The patient's HMO (health maintenance organization) is most likely to approve this type of treatment if: Select one: A. Dr. Emm will be providing services at the in-patient facility. B. Dr. Emm is a licensed psychologist. C. the patient's presenting problem has become chronic. D. the patient has recently shown a decline in functioning.

Correct Answer is: D Policies regarding in-patient treatment vary from HMO to HMO. However, most emphasize the patient's level of functioning and consider approval of in-patient treatment when there has been a recent decline in functioning.

A psychologist in a rural area is referred a 17-year-old male who has been abusing OxyContin. Although the psychologist is trained and experienced in the treatment of substance abuse, she has not received training in, nor experience with, OxyContin abuse. The psychologist should: Select one: A. refer the patient to a psychologist outside the area who is experienced in the treatment of OxyContin abuse B. agree to treat the patient after attending a workshop on the treatment of OxyContin abuse C. refer the patient for inpatient detoxification D. provide the patient with treatment and read relevant literature

Correct Answer is: D Psychologists have an ethical responsibility to practice within the boundaries of their competence. However, according to APA's Ethics Standard 2.01(d), "When psychologists are asked to provide services to individuals for whom appropriate mental health services are not available and for which psychologists have not obtained the competence necessary, psychologists with closely related prior training or experience may provide such services in order to ensure that services are not denied if they make a reasonable effort to obtain the competence required by using relevant research, training, consultation, or study." It should not, therefore, be necessary to refer the patient to a provider out of the area. It also would not be in the patient's best interest to postpone treatment until the psychologist can attend a workshop on the subject. And, although OxyContin abuse is serious and potentially fatal, it generally would not require inpatient detoxification unless the abuse progressed to dependence.

You are a psychologist at a state mental hospital. One of your colleagues, a psychiatrist, tells you that one of his clients has reported to him that he has sexually abused a child. The perpetrator still has access to the child. The psychiatrist says that he is not going to report the abuse because the client expressed remorse, promised not to do it again, and promised to continue treatment. Furthermore, he is widely-known and highly respected in the community. The psychiatrist adamantly refuses to report the child abuse. You should: Select one: A. discuss the matter further with your colleague and encourage him to make a report to the appropriate child abuse reporting agency. B. file a complaint against your colleague with APA or state licensing board. C. contact the appropriate child abuse reporting agency and provide them with information about the abuse, but refuse to give the colleague's name. D. contact the appropriate child abuse reporting agency and provide them with information about the abuse and with the colleague's name.

Correct Answer is: D Psychologists, like most health professionals are mandated reporters. In this instance, since the primary reporter refuses to make the report, you would be mandated to report the suspected child abuse to the appropriate child abuse reporting agency. You would also be obligated to report the identity of the nonreporter to that child abuse reporting agency.

You are completing a court-ordered evaluation in order to determine the person's competency to stand trial and during the administration of the MMPI-2 your client jumps out of his seat and shouts "OK, OK, I killed her!" Now you are Select one: A. obligated to include this information in your report; it is a court-ordered report. B. obligated to include this information; privilege is waived because the mental status is part of the defense. C. obligated to include this information; privilege is waived because you know have a Tarasoff situation. D. not obligated to include the confessional information.

Correct Answer is: D Since the question does not indicate that the client has waived consent, the law protects the defendant at this juncture. He is protected by laws that prohibit the use of evaluation data to determine the defendant's guilt. The best course of action is to not include any incriminating evidence in the competency report, but instead only information relevant to the defendant's competence

A therapist whose client has filed an ethics complaint against him receives a request from the APA ethics committee for records pertaining to his work with the complainant. The therapist refuses to supply the committee with the records. Which of the following statements is most true of this situation? Select one: A. the therapist acted ethically. B. the therapist acted ethically unless he possesses a written release of information from the client. C. the therapist should have consulted with an attorney first. D. the therapist is now liable for an additional ethics violation.

Correct Answer is: D Standard 1.06 of APA's ethical standards states "Psychologists cooperate in ethics investigations, proceedings, and resulting requirements of the APA or any affiliated state psychological association to which they belong...Failure to cooperate is itself an ethics violation." In other words, failure to release the records is itself an ethical violation. the therapist acted ethically unless he possesses a written release of information from the client. If you chose this option, you were correct to be concerned about confidentiality issues in situations like this. However, the client could not have proceeded with his complaint unless he provided a release of information with it.

A psychologist sees clients at a facility where fees are capitated. What does this mean? Select one: A. providers are paid a fee each time a service is performed B. clients are billed according to their ability to pay C. clients are required to meet a deductible before their insurance will pay D. providers receive a fixed dollar amount over a specific period of time to cover the service needs of a fixed number of clients

Correct Answer is: D The term "capitation" refers to a fixed amount of money paid per person, not by the visit or procedure. Managed care companies usually express capitation in terms of cost per member per month. providers receive a fixed dollar amount over a specific period of time to cover the service needs of a fixed number of clients As stated in this response, providers are paid a specific dollar amount, for a specific time period, to cover the service needs of a specific number of people. If a provider exceeds his or her capitated payment, he or she may not be able to cover his or her costs and, consequently, may limit his or her services.

Fee arrangements should be made with clients Select one: A. at the beginning of treatment. B. at the end of the first session. C. on the phone before the first session. D. as soon as possible.

Correct Answer is: D This is an example of a question with more than one good answer. The other choices may all be acceptable, depending on the circumstances. as soon as possible. But because there are a number of acceptable answers to this question, you'd hope to find one general answer, such as this one, that covers most or all of the specific possible options. More importantly, of the choices listed, the correct answer is most consistent with the language of APA's Ethical Standard 6.04(a), which states: "As early as is feasible in a professional or scientific relationship, psychologists and recipients of psychological services reach an agreement specifying compensation and billing arrangements.

You are seeing a client whom you discover, in your third session with him, is the second cousin of your child's barber. In this situation, your ethical obligation is to Select one: A. terminate therapy as soon as possible because multiple relationships are prohibited. B. immediately refer the client to another therapist. C. continue seeing the client for a few more sessions and refer the client out when appropriate. D. continue to see the client.

Correct Answer is: D This question is about multiple relationships, or relationships where a psychologist sees a client professionally but is also involved in some other kind of relationship with him or her, such as a business or personal relationship. The point of this particular question is that not all multiple relationships are unethical. The ethical standard applicable to multiple relationships, standard 3.05, states that psychologists refrain "from entering into a multiple relationship if [it] ...could reasonably be expected to impair the psychologist's objectivity, competence, or effectiveness"¦or otherwise risk exploitation or harm." It also states "Multiple relationships that would not reasonably be expected to cause impairment or risk exploitation or harm are not unethical." Though you would need more information to be certain, it is most reasonable to assume that a professional relationship with the second cousin of your child's barber falls into the latter category of multiple relationships that would not likely cause harm or risk. If the question stated that your client was (for example) your husband's boss, your obligations would be different, because in that situation, you could expect that your objectivity would be impaired. In that situation, you would not take the case to begin with if you knew in advance that the client was your husband's boss. If you found this out in the course of therapy, you would have to, according to the same ethical standard, "take reasonable steps to resolve [the relationship] with due regard for the best interests of the affected person and maximal compliance with the Ethics Code."

A psychological researcher studying the effects of frustrated expectations promises to pay research participants $50.00 for their participation in the study when in reality he does not plan to pay them anything. This deception is Select one: A. ethical as long as it is justified by the subject's prospective scientific value. B. ethical as long as subjects are informed that they have the right to withdraw at any time. C. ethical as long as subjects are wealthy enough to not suffer emotional harm due to not receiving the money. D. unethical.

Correct Answer is: D This question requires you to consider the ethical standards related to both deception and informed consent to research. APA's Ethical Principles and Code of Conduct states that "Psychologists do not deceive prospective participants about research that is reasonably expected to cause physical pain or severe emotional distress." This standard alone would probably bar the research described by the question, though perhaps one can argue that withholding a promised payment of $50.00 would not necessarily cause "severe emotional distress." However, the informed consent requirement states that psychologists must inform research participants about "reasonably foreseeable factors that may be expected to influence their willingness to participate" and "incentives for participation." Therefore, deceiving subjects about how much they will be paid for research participation is unethical.

A client of yours has recently filed a suit against her employer for sex discrimination. You receive a subpoena from the employer's attorney requesting you to appear at a deposition and to bring the client's file. When you contact the client, she tells you she doesn't want you to provide any information about the things she has talked about in therapy. Your best course of action is to: Select one: A. provide the requested information since privilege is waived in this situation. B. provide only the information you believe is relevant to the case. C. appear at the deposition as requested but refuse to provide information about the woman. D. discuss the situation with the employer's attorney before the date of the deposition.

Correct Answer is: D This situation is a common question on the exam, and resolving the issue is not always simple. Certainly, a psychologist could assert the privilege at a deposition but a better initial step would be to contact the employer's attorney to discuss the possibility of withdrawing the request for information (or limiting the kind of information that is required) before the deposition.

According to APA's Standards for Educational and Psychological Testing, test results should be reported Select one: A. only to professionals trained in the interpretation of psychological tests. B. using only raw scores. C. using only confidence intervals or percentile bands. D. in clear and simple language.

Correct Answer is: D To reason this one out, you don't really have to be familiar with the language of the applicable standard, but here it is anyway: "Those responsible for testing programs should provide appropriate interpretations when test score information is released to students, parents, legal representatives, teachers, or the media. The interpretations should describe in simple language what the test covers, what scores mean, common misinterpretations of test scores, and how scores will be used" (standard 15.10). using only confidence intervals or percentile bands. Other standards require that the standard error of measurement be reported so confidence intervals or percentile bands can be constructed; however, the word "only" makes this option incorrect.

A psychologist is contacted by email by a woman seeking online therapy to become more motivated to make some positive life changes. The psychologist agrees to provide the e-therapy and therefore should do which of the following in terms of charging this client for his services? Select one: A. have the client e-mail her credit card information to him before providing services B. charge per e-mail exchange, based on the fee that the client agrees to beforehand C. charge per minute, based on the fee that the client agrees to beforehand D. establish the fee with the client before providing his services

Correct Answer is: D charge per e-mail exchange, based on the fee that the client agrees to beforehand charge per minute, based on the fee that the client agrees to beforehand Although these two responses are acceptable, only the correct response describes something that any e-therapist "should" do. One practice standard for psychologists offering services online is to state their fee structure clearly - clearly stating the amount of their fee on their Web page and communicating with clients about the fee before providing any services. Many e-therapists do what is described in "charge per e-mail exchange, based on the fee that the client agrees to beforehand" - they charge their hourly fee and charge for the time they spend reading and answering each e-mail from a client. have the client e-mail her credit card information to him before providing services An e-therapist can have clients pay by mail, using a check or money order, or accept payment via a credit card. An e-therapist who accepts credit cards, however, must allow payment via a secure server or give the client a phone or fax number to call with his credit card inform

The use of deception in research is acceptable: Select one: A. whenever alternative procedures are not feasible B. if the alternative procedures are too costly C. as long as the deception is not about significant aspects that would affect their willingness to participate D. if is justified by the study's prospective value; if equally effective alternatives are not feasible; and participants are debriefed as soon as possible

Correct Answer is: D if it is justified by the study's prospective value; if equally effective alternatives are not feasible; and participants are debriefed as soon as possible Although the other choices are not incorrect, this choice is the most complete and therefore, the best answer.


Ensembles d'études connexes

ECO 202 Final (study this + other 2)

View Set

GLBA - Gramm-Leach Bliley Act for Payday lenders

View Set

060 - Chapter 60 - Basics of Demographics

View Set

Unit 3 - Chapter 18 - Introduction to Thermodynamics

View Set

Medical Terminology Lesson 8: Exercise 2

View Set

Wolfpacc-Vitamins, Minerals & General Principals

View Set

Unit 8 Content Questions 5.D/6.E

View Set